home *** CD-ROM | disk | FTP | other *** search
/ Internet Info 1994 March / Internet Info CD-ROM (Walnut Creek) (March 1994).iso / answers / rec / puzzles / archive / decision < prev    next >
Text File  |  1993-08-17  |  60KB  |  1,265 lines

  1. Newsgroups: rec.puzzles,news.answers,rec.answers
  2. Path: senator-bedfellow.mit.edu!bloom-beacon.mit.edu!gatech!europa.eng.gtefsd.com!uunet!questrel!chris
  3. From: chris@questrel.com (Chris Cole)
  4. Subject: rec.puzzles Archive (decision), part 12 of 35
  5. Message-ID: <puzzles/archive/decision_745653851@questrel.com>
  6. Followup-To: rec.puzzles
  7. Summary: This is part of an archive of questions
  8.  and answers that may be of interest to
  9.  puzzle enthusiasts.
  10.  Part 1 contains the index to the archive.
  11.  Read the rec.puzzles FAQ for more information.
  12. Sender: chris@questrel.com (Chris Cole)
  13. Reply-To: archive-comment@questrel.com
  14. Organization: Questrel, Inc.
  15. References: <puzzles/archive/Instructions_745653851@questrel.com>
  16. Date: Wed, 18 Aug 1993 06:05:17 GMT
  17. Approved: news-answers-request@MIT.Edu
  18. Expires: Thu, 1 Sep 1994 06:04:11 GMT
  19. Lines: 1243
  20. Xref: senator-bedfellow.mit.edu rec.puzzles:25000 news.answers:11520 rec.answers:1920
  21.  
  22. Archive-name: puzzles/archive/decision
  23. Last-modified: 17 Aug 1993
  24. Version: 4
  25.  
  26.  
  27. ==> decision/allais.p <==
  28. The Allais Paradox involves the choice between two alternatives:
  29.  
  30.     A. 89% chance of an unknown amount
  31.        10% chance of $1 million
  32.        1% chance of $1 million
  33.     B. 89% chance of an unknown amount (the same amount as in A)
  34.        10% chance of $2.5 million
  35.        1% chance of nothing
  36.  
  37. What is the rational choice?  Does this choice remain the same if the
  38. unknown amount is $1 million?  If it is nothing?
  39.  
  40. ==> decision/allais.s <==
  41. This is "Allais' Paradox".
  42.  
  43. Which choice is rational depends upon the subjective value of money.
  44. Many people are risk averse, and prefer the better chance of $1
  45. million of option A.  This choice is firm when the unknown amount is
  46. $1 million, but seems to waver as the amount falls to nothing.  In the
  47. latter case, the risk averse person favors B because there is not much
  48. difference between 10% and 11%, but there is a big difference between
  49. $1 million and $2.5 million.
  50.  
  51. Thus the choice between A and B depends upon the unknown amount, even
  52. though it is the same unknown amount independent of the choice.  This
  53. violates the "independence axiom" that rational choice between two
  54. alternatives should depend only upon how those two alternatives
  55. differ.
  56.  
  57. However, if the amounts involved in the problem are reduced to tens of
  58. dollars instead of millions of dollars, people's behavior tends to
  59. fall back in line with the axioms of rational choice.  People tend to
  60. choose option B regardless of the unknown amount.  Perhaps when
  61. presented with such huge numbers, people begin to calculate
  62. qualitatively.  For example, if the unknown amount is $1 million the
  63. options are:
  64.  
  65.     A. a fortune, guaranteed
  66.     B. a fortune, almost guaranteed
  67.        a tiny chance of nothing
  68.  
  69. Then the choice of A is rational.  However, if the unknown amount is
  70. nothing, the options are:
  71.  
  72.     A. small chance of a fortune ($1 million)
  73.        large chance of nothing
  74.     B. small chance of a larger fortune ($2.5 million)
  75.        large chance of nothing
  76.  
  77. In this case, the choice of B is rational.  The Allais Paradox then
  78. results from the limited ability to rationally calculate with such
  79. unusual quantities.  The brain is not a calculator and rational
  80. calculations may rely on things like training, experience, and
  81. analogy, none of which would be help in this case.  This hypothesis
  82. could be tested by studying the correlation between paradoxical
  83. behavior and "unusualness" of the amounts involved.
  84.  
  85. If this explanation is correct, then the Paradox amounts to little
  86. more than the observation that the brain is an imperfect rational
  87. engine.
  88.  
  89. ==> decision/division.p <==
  90. N-Person Fair Division
  91.  
  92. If two people want to divide a pie but do not trust each other, they can
  93. still ensure that each gets a fair share by using the technique that one
  94. person cuts and the other person chooses. Generalize this technique
  95. to more than two people. Take care to ensure that no one can be cheated
  96. by a coalition of the others.
  97.  
  98.  
  99. ==> decision/division.s <==
  100. N-Person Fair Division
  101.  
  102. Number the people from 1 to N. Person 1 cuts off a piece of the pie.
  103. Person 2 can either diminish the size of the cut off piece or pass.
  104. The same for persons 3 through N. The last person to touch the piece
  105. must take it and is removed from the process. Repeat this procedure
  106. with the remaining N - 1 people, until everyone has a piece.
  107.  
  108. References:
  109. Luce and Raiffa, "Games and Decisions", Wiley, 1957, p. 366
  110. Kenneth Rebman, "How To Get (At Least) A Fair Share of the Cake",
  111.     in Mathematical Plums, Ross Honsberger, ed, Dolciani Mathematical
  112.     Expostions Number 4, published by the MAA.
  113.  
  114. There is a cute result in combinatorics called the Marriage Theorem.
  115. A village has n men and n women, such that for all 0 < k <= n and for any
  116. set of k men there are at least k women, each of whom is in love with at least
  117. one of the k men.  All of the men are in love with all of the women :-}.
  118. The theorem asserts that there is a way to arrange the village into n
  119. monogamous couplings.
  120.  
  121. The Marriage Theorem can be applied to the Fair Pie-Cutting Problem.
  122.  
  123. One player cuts the pie into n pieces.  Each of the players labels
  124. some non-null subset of the pieces as acceptable to him.  For reasons
  125. given below he should "accept" each piece of size > 1/n, not just the
  126. best piece(s).  The pie-cutter is required to "accept" all of the pieces.
  127.  
  128. Given a set S of players let S' denote the set of pie-pieces
  129. acceptable to at least one player in S.  Let t be the size of the largest
  130. set (T) of players satisfying  |T| > |T'|.  If there is no such set, the
  131. Marriage Theorem can be applied directly.  Since the pie-cutter accepts
  132. every piece we know that  t < n.
  133.  
  134. Choose  |T| - |T'|  pieces at random from outside T', glue them
  135. together with the pieces in T' and let the players in T repeat the game
  136. with this smaller (t/n)-size pie.  This is fair since they all rejected
  137. the other n-t pieces, so they believe this pie is larger than t/n.
  138.  
  139. The remaining n-t players can each be assigned one of the remaining
  140. n-t pie-pieces without further ado due to the Marriage Theorem.  (Otherwise
  141. the set T above was not maximal.)
  142.  
  143. The problem of getting not just a fair solution, but an envy-free solution,
  144. is not solved.  A reference to this problem:
  145. David Gale, "Dividing a Cake," in Mathematical Entertainments,
  146.     Mathematical Intelligencer, Vol. 15, No. 1, Winter 1993, p. 50,
  147.     contains references to work by Steven Breams and Alan Taylor.
  148.  
  149. ==> decision/dowry.p <==
  150. Sultan's Dowry
  151.  
  152. A sultan has granted a commoner a chance to marry one of his hundred
  153. daughters. The commoner will be presented the daughters one at a time.
  154. When a daughter is presented, the commoner will be told the daughter's
  155. dowry. The commoner has only one chance to accept or reject each
  156. daughter; he cannot return to a previously rejected daughter.
  157. The sultan's catch is that the commoner may only marry the daughter with
  158. the highest dowry. What is the commoner's best strategy assuming
  159. he knows nothing about the distribution of dowries?
  160.  
  161.  
  162. ==> decision/dowry.s <==
  163. Solution
  164.  
  165. Since the commoner knows nothing about the distribution of the dowries,
  166. the best strategy is to wait until a certain number of daughters have
  167. been presented then pick the highest dowry thereafter. The exact number to
  168. skip is determined by the condition that the odds that the highest dowry
  169. has already been seen is just greater than the odds that it remains to be
  170. seen AND THAT IF IT IS SEEN IT WILL BE PICKED. This amounts to finding the
  171. smallest x such that:
  172.     x/n > x/n * (1/(x+1) + ... + 1/(n-1)).
  173. Working out the math for n=100 and calculating the probability gives:
  174. The commoner should wait until he has seen 37 of the daughters,
  175. then pick the first daughter with a dowry that is bigger than any
  176. preceding dowry. With this strategy, his odds of choosing the daughter
  177. with the highest dowry are surprisingly high: about 37%.
  178. (cf. F. Mosteller, "Fifty Challenging Problems in Probability with Solutions",
  179. Addison-Wesley, 1965, #47; "Mathematical Plums", edited by Ross Honsberger,
  180. pp. 104-110)
  181.  
  182. Here's a twist on the sultan's dowry problem I hope hasn't been posted yet.
  183. I became interested in an iterated version of this problem, which goes as
  184. follows:
  185.  
  186. There's a long line of suitors outside the sultan's palace, and one by one
  187. they come in.  If a suitor gets the right girl, he marries her, as before.
  188. Unfortunately (for the suitor, at least), if he doesn't, he gets his head
  189. lopped off, and the next suitor comes in.
  190.  
  191. Anyway, the first few dozen guys all know their probability theory, so they
  192. know that the best strategy is to skip the first 37 girls, and then pick
  193. the first girl who is the best up to that point.  Alas, each one assumes
  194. that he's the only one who knows that strategy, so one by one, these few
  195. dozen guys get their heads lopped off.
  196.  
  197. After the 49th head has just rolled down the hill, and the sultan's vizier
  198. has just cried out, "Next!" the next guy in line says, "This isn't working
  199. out.  We might all be doing the same thing.  It doesn't hurt any of us to
  200. tell the rest what strategy we'll be using, so that none of us sets out to
  201. pick the same girl over and over again.  I might as well just tell you,
  202. though, that I'm going to get her!  I know this great strategy where you
  203. skip the first 37 blah blah blah..."  Naturally, a few moments later, head
  204. number 50 comes rolling down.
  205.  
  206. "Next!" cries the vizier.
  207.  
  208. Well, suitor number 51 is in a quandary.  He's all set to skip 37, etc, etc,
  209. except now he knows, that's not the right strategy.  But he doesn't know if
  210. the last guy skipped the right girl because she was in the first 37, or if
  211. he didn't meet her yet because he stopped too early.
  212.  
  213. QUESTION 1: What is his best strategy?
  214.  
  215. ANSWER 1: His best strategy is:
  216.  
  217.     "Skip the first 14.  Take the first girl in [15,37] who is better
  218.     than the first 14.  If there isn't one, take the SECOND girl in [38,100]
  219.     who is the best up to that point."
  220.  
  221. Unfortunately, head number 51 rolls down the hill.  "Next!" cries the vizier,
  222. who is getting a little hoarse, and wishes he didn't have this job.
  223.  
  224. QUESTION 2: What is suitor number 52's best strategy?
  225.  
  226. ANSWER 2: His best strategy is:
  227.  
  228.     "Skip the first 5.  Take the first girl in [6,14] who is better than
  229.     the first 5.  If there isn't one, take the SECOND girl in [15,37]
  230.     who is the best up to that point.  If there isn't one, take the THIRD
  231.     girl in [38,100] who is the best up to that point."
  232.  
  233. By the end of the day, of course, a wedding will be set, won't it?
  234.  
  235. MORE QUESTIONS: If each suitor uses the best strategy at that point, how
  236.     many suitors will it take before the right girl is certain to be found?
  237.     Does each succeeding suitor always have a better chance of winning
  238.     than the preceding one?
  239.     
  240. SPECULATION: The last strategy is "Pick the last girl."  Its probability
  241.     of success is 1.  And it is strategy number 100.  (The corresponding
  242.     conditions hold for 3, 4, and 5 daughters.)
  243.  
  244. Does anyone have any observations on this one?
  245.  
  246. byron elbows
  247. (mail to brian@cs.ucla.edu)
  248.  
  249.  
  250.  
  251. ==> decision/envelope.p <==
  252. Someone has prepared two envelopes containing money.  One contains twice as
  253. much money as the other.  You have decided to pick one envelope, but then the
  254. following argument occurs to you:  Suppose my chosen envelope contains $X,
  255. then the other envelope either contains $X/2 or $2X.  Both cases are
  256. equally likely, so my expectation if I take the other envelope is
  257. .5 * $X/2 + .5 * $2X = $1.25X, which is higher than my current $X, so I
  258. should change my mind and take the other envelope.  But then I can apply the
  259. argument all over again.  Something is wrong here!  Where did I go wrong?
  260.  
  261. In a variant of this problem, you are allowed to peek into the envelope
  262. you chose before finally settling on it.  Suppose that when you peek you
  263. see $100.  Should you switch now?
  264.  
  265. ==> decision/envelope.s <==
  266. Let's follow the argument carefully, substituting real numbers for
  267. variables, to see where we went wrong.  In the following, we will assume
  268. the envelopes contain $100 and $200.  We will consider the two equally
  269. likely cases separately, then average the results.
  270.  
  271. First, take the case that X=$100.
  272.  
  273. "I have $100 in my hand.  If I exchange I get $200.  The value of the exchange
  274. is $200.  The value from not exchanging is $100.  Therefore, I gain $100
  275. by exchanging."
  276.  
  277. Second, take the case that X=$200.
  278.  
  279. "I have $200 in my hand.  If I exchange I get $100.  The value of the exchange
  280. is $100.  The value from not exchanging is $200.  Therefore, I lose $100
  281. by exchanging."
  282.  
  283. Now, averaging the two cases, I see that the expected gain is zero.
  284.  
  285. So where is the slip up?  In one case, switching gets X/2 ($100), in the
  286. other case, switching gets 2X ($200), but X is different in the two
  287. cases, and I can't simply average the two different X's to get 1.25X.
  288. I can average the two numbers ($100 and $200) to get $150, the expected
  289. value of switching, which is also the expected value of not switching,
  290. but I cannot under any circumstances average X/2 and 2X.
  291.  
  292. This is a classic case of confusing variables with constants.
  293.  
  294. OK, so let's consider the case in which I looked into the envelope and
  295. found that it contained $100.  This pins down what X is: a constant.
  296.  
  297. Now the argument is that the odds of $50 is .5 and the odds of $200
  298. is .5, so the expected value of switching is $125, so we should switch.
  299. However, the only way the odds of $50 could be .5 and the odds of $200
  300. could be .5 is if all integer values are equally likely.  But any
  301. probability distribution that is finite and equal for all integers
  302. would sum to infinity, not one as it must to be a probability distribution.
  303. Thus, the assumption of equal likelihood for all integer values is
  304. self-contradictory, and leads to the invalid proof that you should
  305. always switch.  This is reminiscent of the plethora of proofs that 0=1;
  306. they always involve some illegitimate assumption, such as the validity
  307. of division by zero.
  308.  
  309. Limiting the maximum value in the envelopes removes the self-contradiction
  310. and the argument for switching.  Let's see how this works.
  311.  
  312. Suppose all amounts up to $1 trillion were equally likely to be
  313. found in the first envelope, and all amounts beyond that would never
  314. appear.  Then for small amounts one should indeed switch, but not for
  315. amounts above $500 billion.  The strategy of always switching would pay
  316. off for most reasonable amounts but would lead to disastrous losses for
  317. large amounts, and the two would balance each other out.
  318.  
  319. For those who would prefer to see this worked out in detail:
  320. Assume the smaller envelope is uniform on [$0,$M], for some value
  321. of $M.  What is the expectation value of always switching?  A quarter of
  322. the time $100 >= $M (i.e. 50% chance $X is in [$M/2,$M] and 50% chance
  323. the larger envelope is chosen).  In this case the expected switching
  324. gain is -$50 (a loss).  Thus overall the always switch policy has an
  325. expected (relative to $100) gain of (3/4)*$50 + (1/4)*(-$50) = $25.
  326. However the expected absolute gain (in terms of M) is:
  327.   / M 
  328.   |    g f(g) dg,   [ where f(g) = (1/2)*Uniform[0,M)(g) +
  329.   /-M                              (1/2)*Uniform(-M,0](g). ]
  330.  
  331.   = 0.  QED.
  332.  
  333. OK, so always switching is not the optimal switching strategy.  Surely
  334. there must be some strategy that takes advantage of the fact that we
  335. looked into the envelope and we know something we did not know before
  336. we looked.
  337.  
  338. Well, if we know the maximum value $M that can be in the smaller envelope,
  339. then the optimal decision criterion is to switch if $100 < $M, otherwise stick.
  340. The reason for the stick case is straightforward. The reason for the
  341. switch case is due to the pdf of the smaller envelope being twice as
  342. high as that of the larger envelope over the range [0,$M). That is, the
  343. expected gain in switching is (2/3)*$100 + (1/3)*(-$50) = $50.
  344.  
  345. What if we do not know the maximum value of the pdf?  You can exploit
  346. the "test value" technique to improve your chances.  The trick here is
  347. to pick a test value T.  If the amount in the envelope is less than the
  348. test value, switch; if it is more, do not.  This works in that if T happens
  349. to be in the range [M,2M] you will make the correct decision.  Therefore,
  350. assuming the unknown pdf is uniform on [0,M], you are slightly better off
  351. with this technique.
  352.  
  353. Of course, the pdf may not even be uniform, so the "test value" technique
  354. may not offer much of an advantage.  If you are allowed to play the game
  355. repeatedly, you can estimate the pdf, but that is another story...
  356.  
  357. ==> decision/exchange.p <==
  358. At one time, the Canadian and US dollars were discounted by 10 cents on
  359. each side of the border (i.e., a Canadian dollar was worth 90 US cents
  360. in the US, and a US dollar was worth 90 Canadian cents in Canada).  A
  361. man walks into a bar on the US side of the border, orders 10 US cents
  362. worth of beer, pays with a US dollar and receives a Canadian dollar in
  363. change.  He then walks across the border to Canada, orders 10 Canadian
  364. cents worth of beer, pays with a Canadian dollar and receives a US
  365. dollar in change.  He continues this throughout the day, and ends up
  366. dead drunk with the original dollar in his pocket.
  367.  
  368. Who pays for the drinks?
  369.  
  370. ==> decision/exchange.s <==
  371. The man paid for all the drinks.  But, you say, he ended up with the
  372. same amount of money that he started with!  However, as he transported
  373. Canadian dollars into Canada and US dollars into the US, he performed
  374. "economic work" by moving the currency to a location where it was in
  375. greater demand (and thus valued higher).  The earnings from this work
  376. were spent on the drinks.
  377.  
  378. Note that he can only continue to do this until the Canadian bar runs
  379. out of US dollars, or the US bar runs out of Canadian dollars, i.e.,
  380. until he runs out of "work" to do.
  381.  
  382. ==> decision/high.or.low.p <==
  383. I pick two numbers, randomly, and tell you one of them. You are supposed
  384. to guess whether this is the lower or higher one of the two numbers I
  385. picked. Can you come up with a method of guessing that does better than
  386. picking the response "low" or "high" randomly (i.e. probability to guess
  387. right > .5) ?
  388.  
  389. ==> decision/high.or.low.s <==
  390. Pick any cumulative probability function P(x) such that a > b ==> P(a)
  391. > P(b).  Now if the number shown is y, guess "low" with probability
  392. P(y) and "high" with probability 1-P(y).  This strategy yields a
  393. probability of > 1/2 of winning since the probability of being correct
  394. is 1/2*( (1-P(a)) + P(b) ) = 1/2 + (P(b)-P(a)), which is > 1/2 by
  395. assumption.
  396.  
  397. ==> decision/monty.hall.p <==
  398. You are a participant on "Let's Make a Deal." Monty Hall shows you
  399. three closed doors.  He tells you that two of the closed doors have a
  400. goat behind them and that one of the doors has a new car behind it.
  401. You pick one door, but before you open it, Monty opens one of the two
  402. remaining doors and shows that it hides a goat.  He then offers you a
  403. chance to switch doors with the remaining closed door.  Is it to your
  404. advantage to do so?
  405.  
  406. ==> decision/monty.hall.s <==
  407. Under reasonable assumptions about Monty Hall's motivation, your chance
  408. of picking the car doubles when you switch.
  409.  
  410. The problem is confusing for two reasons:  first,  there are hidden
  411. assumptions about Monty's motivation that cloud the issue; and second,
  412. novice probability students do not see that the opening of the door
  413. gave them any new information.
  414.  
  415. Monty can have one of three basic motives:
  416. 1.  He randomly opens doors.
  417. 2.  He always opens the door he knows contains nothing.
  418. 3.  He only opens a door when the contestant has picked the grand prize.
  419.  
  420. These result in very different strategies:
  421. 1.  No improvement when switching.
  422. 2.  Double your odds by switching.
  423. 3.  Don't switch!
  424.  
  425. Most people, myself included, think that (2) is the intended
  426. interpretation of Monty's motive.  Interviews with Monty Hall
  427. indicate that he did indeed try to lure the contestant who had picked
  428. the car with cash incentives to switch.  However, if Monty always
  429. adopted this strategy, contestants would soon learn never to switch,
  430. so one presumes that occasionally Monty offered another door even when
  431. the contestant had picked a goat.  At any rate, analyzing the problem
  432. with this strategy is difficult, since it requires knowing something
  433. about Monty's probability of bluffing.
  434.  
  435. A good way to see that Monty is giving you information by opening doors
  436. that he knows are valueless is to increase the number of doors from
  437. three to 100.  If there are 100 doors, and Monty shows that 98 of them
  438. are valueless, isn't it pretty clear that the chance the prize is behind
  439. the remaining door is 99/100?
  440.  
  441. The original Monty Hall problem (and solution) appears to be due to
  442. Steve Selvin, and appears in American Statistician, Feb 1975, V. 29,
  443. No. 1, p. 67 under the title ``A Problem in Probability.''  It should
  444. be of no surprise to readers of this group that he received several
  445. letters contesting the accuracy of his solution, so he responded two
  446. issues later (American Statistician, Aug 1975, V. 29, No. 3, p. 134).
  447. However, the principles that underlie the problem date back at least to
  448. the fifties, and probably are timeless.  See the references below for
  449. details.
  450.  
  451. Reference (too numerous to mention, but these contain bibliographies):
  452.     Leonard Gillman, "The Car and the Goats", AMM 99:1 (Jan 1992), p. 3
  453.     Ed Barbeau, "The Problem of the Car and Goats", CMJ 24:2 (Mar 1993), p. 149
  454.  
  455. The second reference contains a list of equivalent or related problems.
  456.  
  457. ==> decision/newcomb.p <==
  458. Newcomb's Problem
  459.  
  460. A being put one thousand dollars in box A and either zero or one million
  461. dollars in box B and presents you with two choices:
  462.     (1) Open box B only.
  463.     (2) Open both box A and B.
  464. The being put money in box B only if it predicted you will choose option (1).
  465. The being put nothing in box B if it predicted you will do anything other than
  466. choose option (1) (including choosing option (2), flipping a coin, etc.).
  467.  
  468. Assuming that you have never known the being to be wrong in predicting your
  469. actions, which option should you choose to maximize the amount of money you
  470. get?
  471.  
  472.  
  473. ==> decision/newcomb.s <==
  474. This is "Newcomb's Paradox".
  475.  
  476. You are presented with two boxes: one certainly contains $1000 and the
  477. other might contain $1 million.  You can either take one box or both.
  478. You cannot change what is in the boxes.  Therefore, to maximize your
  479. gain you should take both boxes.
  480.  
  481. However, it might be argued that you can change the probability that
  482. the $1 million is there.  Since there is no way to change whether the
  483. million is in the box or not, what does it mean that you can change
  484. the probability that the million is in the box?  It means that your
  485. choice is correlated with the state of the box.
  486.  
  487. Events which proceed from a common cause are correlated.  My mental
  488. states lead to my choice and, very probably, to the state of the box.
  489. Therefore my choice and the state of the box are highly correlated.
  490. In this sense, my choice changes the "probability" that the money is
  491. in the box.  However, since your choice cannot change the state of the
  492. box, this correlation is irrelevant.
  493.  
  494. The following argument might be made: your expected gain if you take
  495. both boxes is (nearly) $1000, whereas your expected gain if you take
  496. one box is (nearly) $1 million, therefore you should take one box.
  497. However, this argument is fallacious.  In order to compute the
  498. expected gain, one would use the formulas:
  499.  
  500.     E(take one) =    $0 * P(predict take both | take one) + 
  501.                 $1,000,000 * P(predict take one | take one)
  502.     E(take both) =    $1,000 * P(predict take both | take both) +
  503.                 $1,001,000 * P(predict take one | take both)
  504.  
  505. While you are given that P(do X | predict X) is high, it is not given
  506. that P(predict X | do X) is high.  Indeed, specifying that P(predict X
  507. | do X) is high would be equivalent to specifying that the being could
  508. use magic (or reverse causality) to fill the boxes.  Therefore, the
  509. expected gain from either action cannot be determined from the
  510. information given.
  511.  
  512.  
  513. ==> decision/prisoners.p <==
  514. Three prisoners on death row are told that one of them has been chosen
  515. at random for execution the next day, but the other two are to be
  516. freed.  One privately begs the warden to at least tell him the name of
  517. one other prisoner who will be freed.  The warden relents: 'Susie will
  518. go free.'  Horrified, the first prisoner says that because he is now
  519. one of only two remaining prisoners at risk, his chances of execution
  520. have risen from one-third to one-half!  Should the warden have kept his
  521. mouth shut?
  522.  
  523. ==> decision/prisoners.s <==
  524. Each prisoner had an equal chance of being the one chosen to be
  525. executed.  So we have three cases:
  526.  
  527. Prisoner executed:         A    B    C
  528. Probability of this case: 1/3  1/3  1/3
  529.  
  530. Now, if A is to be executed, the warden will randomly choose either B or C,
  531. and tell A that name.  When B or C is the one to be executed, there is only
  532. one prisoner other than A who will not be executed, and the warden will always
  533. give that name.  So now we have:
  534.  
  535. Prisoner executed:  A    A    B    C
  536. Name given to A:    B    C    C    B
  537. Probability:       1/6  1/6  1/3  1/3
  538.  
  539. We can calculate all this without knowing the warden's answer.
  540. When he tells us B will not be executed, we eliminate the middle two
  541. choices above.  Now, among the two remaining cases, C is twice
  542. as likely as A to be the one executed.  Thus, the probability that
  543. A will be executed is still 1/3, and C's chances are 2/3.
  544.  
  545. ==> decision/red.p <==
  546. I show you a shuffled deck of standard playing cards, one card at a
  547. time.  At any point before I run out of cards, you must say "RED!".
  548. If the next card I show is red (i.e. diamonds or hearts), you win.  We
  549. assume I the "dealer" don't have any control over what the order of
  550. cards is.
  551.  
  552. The question is, what's the best strategy, and what is your
  553. probability of winning ?
  554.  
  555. ==> decision/red.s <==
  556. If a deck has n cards, r red and b black, the best strategy wins
  557. with a probability of r/n.  Thus, you can say "red" on the first card,
  558. the last card, or any other card you wish.
  559. Proof by induction on n.  The statement is clearly true for one-card decks.
  560. Suppose it is true for n-card decks, and add a red card.
  561. I will even allow a nondeterministic strategy, meaning you say "red"
  562. on the first card with probability p.  With probability 1-p,
  563. you watch the first card go by, and then apply the "optimal" strategy
  564. to the remaining n-card deck, since you now know its composition.
  565. The odds of winning are therefore: p * (r+1)/(n+1)  +
  566.         (1-p) * ((r+1)/(n+1) * r/n  +  b/(n+1) * (r+1)/n).
  567. After some algebra, this becomes (r+1)/(n+1) as expected.
  568. Adding a black card yields: p * r/(n+1)  +
  569.         (1-p) * (r/(n+1) * (r-1)/n  +  (b+1)/(n+1) * r/n).
  570. This becomes r/(n+1) as expected.
  571.  
  572. ==> decision/rotating.table.p <==
  573. Four glasses are placed upside down in the four corners of a square
  574. rotating table.  You wish to turn them all in the same direction,
  575. either all up or all down.  You may do so by grasping any two glasses
  576. and, optionally, turning either over.  There are two catches:  you are
  577. blindfolded and the table is spun after each time you touch the
  578. glasses.  Assuming that a bell rings when you have all the glasses up,
  579. how do you do it?
  580.  
  581. ==> decision/rotating.table.s <==
  582. 1.  Turn two adjacent glasses up.
  583. 2.  Turn two diagonal glasses up.
  584. 3.  Pull out two diagonal glasses.  If one is down, turn it up and you're done.
  585.     If not, turn one down and replace.
  586. 4.  Take two adjacent glasses.  Invert them both.
  587. 5.  Take two diagonal glasses.  Invert them both.
  588.  
  589. References
  590.     "Probing the Rotating Table"
  591.     W. T. Laaser and L. Ramshaw
  592.     _The Mathematical Gardner_,
  593.     Wadsworth International, Belmont CA 1981.
  594.  
  595.     ... we will see that such a procedure exists if and
  596.     only if the parameters k and n satisfy the inequality
  597.     k >= (1-1/p)n, where p is the largest prime factor
  598.     of n.
  599.  
  600. The paper mentions (without discussing) two other generalizations:
  601. more than two orientations of the glasses (Graham and Diaconis)
  602. and more symmetries in the table, e.g. those of a cube (Kim).
  603.  
  604. ==> decision/stpetersburg.p <==
  605. What should you be willing to pay to play a game in which the payoff is
  606. calculated as follows:  a coin is flipped until it comes up heads on the
  607. nth toss and the payoff is set at 2^n dollars?
  608.  
  609. ==> decision/stpetersburg.s <==
  610. Classical decision theory says that you should be willing to pay any
  611. amount up to the expected value of the wager.  Let's calculate the
  612. expected value:  The probability of winning at step n is 2^-n, and the
  613. payoff at step n is 2^n, so the sum of the products of the
  614. probabilities and the payoffs is:
  615.  
  616.     E = sum over n (2^-n * 2^n) = sum over n (1) = infinity
  617.  
  618. So you should be willing to pay any amount to play this game.  This is
  619. called the "St. Petersburg Paradox."
  620.  
  621. The classical solution to this problem was given by Bernoulli.  He
  622. noted that people's desire for money is not linear in the amount of
  623. money involved.  In other words, people do not desire $2 million twice
  624. as much as they desire $1 million.  Suppose, for example, that people's
  625. desire for money is a logarithmic function of the amount of money.
  626. Then the expected VALUE of the game is:
  627.  
  628.     E = sum over n (2^-n * C * log(2^n)) = sum over n (2^-n * C' * n) =  C''
  629.  
  630. Here the C's are constants that depend upon the risk aversion of the
  631. player, but at least the expected value is finite.  However, it turns
  632. out that these constants are usually much higher than people are really
  633. willing to pay to play, and in fact it can be shown that any
  634. non-bounded utility function (map from amount of money to value of
  635. money) is prey to a generalization of the St. Petersburg paradox.  So
  636. the classical solution of Bernoulli is only part of the story.
  637.  
  638. The rest of the story lies in the observation that bankrolls are always
  639. finite, and this dramatically reduces the amount you are willing to bet
  640. in the St. Petersburg game.
  641.  
  642. To figure out what would be a fair value to charge for playing the game
  643. we must know the bank's resources.  Assume that the bank has 1 million
  644. dollars (1*K*K = 2^20).  I cannot possibly win more than $1 million
  645. whether I toss 20 tails in a row or 2000.
  646.  
  647. Therefore my expected amount of winning is 
  648.  
  649.     E = sum n up to 20 (2^-n * 2^n) = sum n up to 20 (1) = $20
  650.  
  651. and my expected value of winning is 
  652.  
  653.     E = sum n up to 20 (2^-n * C * log(2^n)) = some small number
  654.  
  655. This is much more in keeping with what people would really pay to
  656. play the game.
  657.  
  658. Incidentally, T.C. Fry suggested this change to the problem in 1928
  659. (see W.W.R. Ball, Mathematical Recreations and Essays, N.Y.: Macmillan,
  660. 1960, pp.  44-45).
  661.  
  662. The problem remains interesting when modified in this way,
  663. for the following reason. For a particular value of the bank's
  664. resources, let
  665.  
  666.       e denote the expected value of the player's winnings; and let
  667.       p denote the probability that the player profits from the game, assuming
  668.         the price of getting into the game is 0.8e (20% discount).
  669.  
  670. Note that the expected value of the player's profit is 0.2e.  Now
  671. let's vary the bank's resources and observe how e and p change.  It
  672. will be seen that as e (and hence the expected value of the profit)
  673. increases, p diminishes.  The more the game is to the player's
  674. advantage in terms of expected value of profit, the less likely it is
  675. that the player will come away with any profit at all.  This 
  676. is mildly counterintuitive.
  677.  
  678. ==> decision/truel.p <==
  679. A, B, and C are to fight a three-cornered pistol duel.  All know that
  680. A's chance of hitting his target is 0.3, C's is 0.5, and B never misses.
  681. They are to fire at their choice of target in succession in the order
  682. A, B, C, cyclically (but a hit man loses further turns and is no longer
  683. shot at) until only one man is left.  What should A's strategy be?
  684.  
  685. ==> decision/truel.s <==
  686. This is problem 20 in Mosteller _Fifty Challenging Problems in Probability_
  687. and it also appears (with an almost identical solution) on page 82 in
  688. Larsen & Marx _An Introduction to Probability and Its Applications_.
  689.  
  690. Here's Mosteller's solution:
  691.  
  692.   A is naturally not feeling cheery about this enterprise.  Having the
  693. first shot he sees that, if he hits C, B will then surely hit him, and
  694. so he is not going to shoot at C.  If he shoots at B and misses him,
  695. then B clearly {I disagree; this is not at all clear!} shoots the more
  696. dangerous C first, and A gets one shot at B with probability 0.3 of
  697. succeeding.  If he misses this time, the less said the better.  On the
  698. other hand, suppose A hits B.  Then C and A shoot alternately until one
  699. hits.  A's chance of winning is (.5)(.3) + (.5)^2(.7)(.3) +
  700. (.5)^3(.7)^2(.3) + ... .  Each term corresponds to a sequence of misses
  701. by both C and A ending with a final hit by A.  Summing the geometric
  702. series we get ... 3/13 < 3/10.  Thus hitting B and finishing off with
  703. C has less probability of winning for A than just missing the first shot.
  704. So A fires his first shot into the ground and then tries to hit B with
  705. his next shot.  C is out of luck.
  706.  
  707. As much as I respect Mosteller, I have some serious problems with this
  708. solution.  If we allow the option of firing into the ground, then if
  709. all fire into the ground with every shot, each will survive with
  710. probability 1.  Now, the argument could be made that a certain
  711. strategy for X that both allows them to survive with probability 1
  712. *and* gives less than a probability of survival of less than 1 for
  713. at least one of their foes would be preferred by X.  However, if
  714. X pulls the trigger and actually hits someone what would the remaining
  715. person, say Y, do?  If P(X hits)=1, clearly Y must try to hit X, since
  716. X firing at Y with intent to hit dominates any other strategy for X.
  717. If P(X hits)<1 and X fires at Y with intent to hit, then
  718. P(Y survives)<1 (since X could have hit Y).  Thus, Y must insure that
  719. X can not follow this strategy by shooting back at X (thus insuring
  720. that P(X survives)<1).  Therefore, I would conclude that the ideal
  721. strategy for all three players, assuming that they are rational and
  722. value survival above killing their enemies, would be to keep firing
  723. into the ground.  If they don't value survival above killing their
  724. enemies (which is the only a priori assumption that I feel can be
  725. safely made in the absence of more information), then the problem
  726. can't be solved unless the function each player is trying to maximize
  727. is explicitly given.
  728. -- 
  729.     -- clong@remus.rutgers.edu (Chris Long)
  730.  
  731. OK - I'll have a go at this.
  732.  
  733. How about the payoff function being 1 if you win the "duel" (i.e. if at some
  734. point you are still standing and both the others have been shot) and 0
  735. otherwise? This should ensure that an infinite sequence of deliberate misses
  736. is not to anyone's advantage. Furthermore, I don't think simple survival
  737. makes a realistic payoff function, since people with such a payoff function
  738. would not get involved in the fight in the first place!
  739.  
  740. [ I.e. I am presupposing a form of irrationality on the part of the
  741.   fighters: they're only interested in survival if they win the duel. Come
  742.   to think of it, this may be quite rational - spending the rest of my life
  743.   firing a gun into the ground would be a very unattractive proposition to
  744.   me :-)
  745. ]
  746.  
  747. Now, denote each position in the game by the list of people left standing,
  748. in the order in which they get their turns (so the initial position is
  749. (A,B,C), and the position after A misses the first shot (B,C,A)). We need to
  750. know the value of each possible position for each person.
  751.  
  752. By definition:
  753.  
  754.     valA(A) = 1            valB(A) = 0            valC(A) = 0
  755.     valA(B) = 0            valB(B) = 1            valC(B) = 0
  756.     valA(C) = 0            valB(C) = 0            valC(C) = 1
  757.  
  758. Consider the two player position (X,Y). An infinite sequence of misses has
  759. value zero to both players, and each player can ensure a positive payoff by
  760. trying to shoot the other player. So both players deliberately missing is a
  761. sub-optimal result for both players. The question is then whether both
  762. players should try to shoot the other first, or whether one should let the
  763. other take the first shot. Since having the first shot is always an
  764. advantage, given that some real shots are going to be fired, both players
  765. should try to shoot the other first. It is then easy to establish that:
  766.  
  767.     valA(A,B) = 3/10       valB(A,B) = 7/10       valC(A,B) = 0
  768.     valA(B,A) = 0          valB(B,A) = 1          valC(B,A) = 0
  769.     valA(B,C) = 0          valB(B,C) = 1          valC(B,C) = 0
  770.     valA(C,B) = 0          valB(C,B) = 5/10       valC(C,B) = 5/10
  771.     valA(C,A) = 3/13       valB(C,A) = 0          valC(C,A) = 10/13
  772.     valA(A,C) = 6/13       valB(A,C) = 0          valC(A,C) = 7/13
  773.  
  774. Now for the three player positions (A,B,C), (B,C,A) and (C,A,B). Again, the
  775. fact that an infinite sequence of misses is sub-optimal for all three
  776. players means that at least one player is going to decide to fire. However,
  777. it is less clear than in the 2 player case that any particular player is
  778. going to fire. In the 2 player case, each player knew that *if* it was
  779. sub-optimal for him to fire, then it was optimal for the other player to
  780. fire *at him* and that he would be at a disadvantage in the ensuing duel
  781. because of not having got the first shot. This is not necessarily true in
  782. the 3 player case.
  783.  
  784. Consider the payoff to A in the position (A,B,C). If he shoots at B, his
  785. expected payoff is:
  786.  
  787.     0.3*valA(C,A) + 0.7*valA(B,C,A) = 9/130 + 0.7*valA(B,C,A)
  788.  
  789. If he shoots at C, his expected payoff is:
  790.  
  791.     0.3*valA(B,A) + 0.7*valA(B,C,A) = 0.7*valA(B,C,A)
  792.  
  793. And if he deliberately misses, his expected payoff is:
  794.  
  795.     valA(B,C,A)
  796.  
  797. Since he tries to maximise his payoff, we can immediately eliminate shooting
  798. at C as a strategy - it is strictly dominated by shooting at B. So A's
  799. expected payoff is:
  800.  
  801.     valA(A,B,C) = MAX(valA(B,C,A), 9/130 + 0.7*valA(B,C,A))
  802.  
  803. A similar argument shows that C's expected payoffs in the (C,A,B) position are:
  804.  
  805.     For shooting at A: 0.5*valC(A,B,C)
  806.     For shooting at B: 35/130 + 0.5*valC(A,B,C)
  807.     For missing:       valC(A,B,C)
  808.  
  809. So C either shoots at B or deliberately misses, and:
  810.  
  811.     valC(C,A,B) = MAX(valC(A,B,C), 35/130 + 0.5*valC(A,B,C))
  812.  
  813. Each player can obtain a positive expected payoff by shooting at one of the
  814. other players, and it is known that an infinite sequence of misses will
  815. result in a zero payoff for all players. So it is known that some player's
  816. strategy must involve shooting at another player rather than deliberately
  817. missing.
  818.  
  819. Now look at this from the point of view of player B. He knows that *if* it
  820. is sub-optimal for him to shoot at another player, then it is optimal for at
  821. least one of the other players to shoot. He also knows that if the other
  822. players choose to shoot, they will shoot *at him*. If he deliberately
  823. misses, therefore, the best that he can hope for is that they miss him and
  824. he is presented with the same situation again. This is clearly less good for
  825. him than getting his shot in first. So in position (B,C,A), he must shoot at
  826. another player rather than deliberately miss.
  827.  
  828. B's expected payoffs are:
  829.  
  830.     For shooting at A: valB(C,B) = 5/10
  831.     For shooting at C: valB(A,B) = 7/10
  832.  
  833. So in position (B,C,A), B shoots at C for an expected payoff of 7/10. This
  834. gives us:
  835.  
  836.     valA(B,C,A) = 3/10     valB(B,C,A) = 7/10     valC(B,C,A) = 0
  837.  
  838. So valA(A,B,C) = MAX(3/10, 9/130 + 21/100) = 3/10, and A's best strategy is
  839. position (A,B,C) is to deliberately miss, giving us:
  840.  
  841.     valA(A,B,C) = 3/10     valB(A,B,C) = 7/10     valC(A,B,C) = 0
  842.  
  843. And finally, valC(C,A,B) = MAX(0, 35/130 + 0) = 7/26, and C's best strategy
  844. in position (C,A,B) is to shoot at B, giving us:
  845.  
  846.     valA(C,A,B) = 57/260   valB(C,A,B) = 133/260  valC(C,A,B) = 7/26
  847.  
  848. I suspect that, with this payoff function, all positions with 3 players can
  849. be resolved. For each player, we can establish that if their correct
  850. strategy is to fire at another player, then it is to fire at whichever of
  851. the other players is more dangerous. The most dangerous of the three players
  852. then finds that he has nothing to lose by firing at the second most
  853. dangerous.
  854.  
  855. Questions:
  856.  
  857. (a) In the general case, what are the optimal strategies for the other two
  858.     players, possibly as functions of the hit probabilities and the cyclic
  859.     order of the three players?
  860.  
  861. (b) What happens in the 4 or more player case?
  862.  
  863.     -- David Seal <dseal@armltd.co.uk>
  864.  
  865. In article <1993Mar25.022459.10269@cs.cornell.edu>, karr@cs.cornell.edu (David Karr) writes:
  866. > The Good, the Bad, and the Ugly are standing at three equidistant
  867.       "P"       "Q"          "R"      -- allow me these alternate names.
  868. > points around a very large circle, about to fight a three-way duel to
  869. > see who gets the treasure.  They all know that the Good hits with
  870. > probability p=.9, the Bad hits with probability q=.7, and the Ugly
  871. > hits with probability r=.5.
  872. > Yes, I know this sounds like decision/truel from the rec.puzzles
  873. > archive.  But here's the difference:
  874. > At some instant, all three will fire simultaneously, each at a target
  875. > of his choice.  Then any who survive that round fire simultaneously
  876. > again, until at most one remains.  Note that there are then four
  877. > possible outcomes: the Good wins, the Bad wins, the Ugly wins, or all
  878. > are killed.
  879.  
  880.     A multi-round multi-person game can get complicated if implicit
  881.     alliances are formed or the players deduce each other's strategies.
  882.     For simplicity let's disallow communication and assume the players
  883.     forget who shot at whom after each round.
  884. > Now the questions:
  885.  
  886.     These are not easy questions, even with the simplifying
  887.     assumptions I've made.
  888.  
  889. > 1. What is each shooter's strategy?
  890.  
  891.     Each player has two possible strategies so there are eight cases
  892.     to consider; unfortunately none of the players has a strictly
  893.     dominant strategy:
  894.  
  895. P aims at Q aims at R aims at    P survival Q survival R survival Noone lives
  896. --------- --------- ---------   ---------- ---------- ---------- -----------
  897.     Q         P         P         0.0649     0.0355     0.7991     0.1005
  898.     Q         P         Q         0.1371     0.0146     0.6966     0.1517
  899.  *  Q         R         P         0.3946     0.0444     0.1470     0.4140
  900.     Q         R         Q         0.8221     0.0026     0.0152     0.1601
  901.     R         P         P         0.0381     0.8221     0.0152     0.1246
  902.  *  R         P         Q         0.1824     0.3443     0.0426     0.4307
  903.     R         R         P         0.1371     0.5342     0.0027     0.3260
  904.     R         R         Q         0.6367     0.0355     0.0008     0.3270
  905.  
  906.     (The similarity of, say, the 4th and 5th lines here looks wrong:
  907.     the intermediate expressions are quite different.  I can't
  908.     explain *why* P_survival(q,r,q) = Q_survival(r,p,p) = 0.8221
  909.     but I *have* double-checked this result.)
  910.  
  911.     If I *know* who my opponents are going to aim at, I should shoot
  912.     at the better shooter if they're both aiming at me or neither is
  913.     aiming at me.  Otherwise I should aim at whoever is *not* aiming
  914.     at me.  There are two equilibrium points (marked "*" above):
  915.         Good aims at Bad; Bad aims at Ugly; Ugly aims at Good.
  916.     and
  917.         Good aims at Ugly; Bad aims at Good; Ugly aims at Bad.
  918.     Here, unlike for zero-sum two-person games, the equilibria
  919.     are *not* equivalent and "solution", if any, may lie elsewhere.
  920.     Perhaps a game-theorist lurking in r.p can offer a better comment.
  921.  
  922.     Note that the probability all three shooters die is highest at
  923.     the equilibria!  This seems rather paradoxical and rather sad :-(
  924. > 2. Who is most likely to survive?
  925.  
  926.     Good, Bad, or Ugly, depending on the strategies.
  927. > 3. Who is least likely to survive?
  928.     Bad or Ugly, depending on the strategies.
  929.  
  930. > 4. Can you change p, q, and r under the constraint p > q > r so that
  931. >    the answers to questions 2 and 3 are reversed?  Which of the six
  932. >    possible permutations of the three shooters is a possible ordering
  933. >    of probability of survival under the constraint p > q > r?
  934.  
  935.     Yes.  Of the six possible survival-probability orderings,
  936.     five can be obtained readily:
  937.         p    q    r    P_surv    Q_surv    R_surv    Order
  938.         ---    ---    ---    ------    ------    ------    -------
  939.         0.255    0.25    0.01    0.408    0.413    0.172    Q P R
  940.         0.26    0.25    0.01    0.412    0.406    0.173    P Q R
  941.         0.75    0.25    0.01    0.675    0.076    0.242    P R Q
  942.         0.505    0.50    0.01    0.325    0.324    0.344    R P Q
  943.         0.505    0.50    0.02    0.314    0.320    0.353    R Q P
  944.  
  945.     Unlike the p=.9, q=.7, r=.5 case we are given, the five cases
  946.     in this table *do* have simple pure solutions: in each case
  947.     p shoots at q, while q and r each shoot at p.  (I've found no
  948.     case with a "simple pure" solution other than this "obvious"
  949.     p aims at q, q aims at p, r aims at p choice.)
  950.  
  951. > 5. Are there any value of p, q, and r for which it is ever in the
  952. >    interest of one of the shooters to fire into the ground?
  953.  
  954.     No.  It can't hurt to shoot at one's stronger opponent.
  955.     This is the easiest of the questions ... but it's still
  956.     not easy enough for me to construct an elegant proof
  957.     in English.
  958.  
  959. > -- David Karr (karr@cs.cornell.edu)
  960.     Speaking of decision/truel, I recall a *very* interesting
  961.     analysis (I *might* have seen it here in rec.puzzles) suggesting
  962.     that the N-person "truel" (N-uel?) has a Cooperative Solution
  963.     (ceasefire) if and only if N = 3.  But I don't see this in the
  964.     FAQL; anyone care to repost it?
  965.  
  966. -- James Allen
  967.  
  968. In article <1993Apr1.123404.18039@vax5.cit.cornell.edu> mkt@vax5.cit.cornell.edu writes:
  969. >In article <1993Mar25.022459.10269@cs.cornell.edu>, karr@cs.cornell.edu (David Karr) writes:
  970. [...]
  971. >> 5. Are there any value of p, q, and r for which it is ever in the
  972. >>    interest of one of the shooters to fire into the ground?
  973. >> 
  974. >     Yes, p=1, q=1, r=1.  The only way for one to survive is to have the other
  975. >     two shoot at eachother.  Shooting at anyone has no effect on ones personal
  976. >     survival.
  977.  
  978. I assume by "has no effect on" you mean "does not improve."
  979.  
  980. >  If all follow the same logic, they will keep shooting into the
  981. >     ground and thus all live.
  982.  
  983. I was very pleased by this answer but I had to think about it.  First
  984. of all, it assumes that continuing the fight forever has a positive
  985. value for each shooter.  My preferred assumption is that it doesn't.
  986. But even if each shooter is simply trying to maximize his probability
  987. of never being shot, I wonder about the "has no effect" statement.
  988.  
  989. Suppose that in round 1 the Good fires into the ground and the Bad
  990. shoots at the Good.  Then the Ugly lives if he shoots the Bad and dies
  991. if he does anything else.  (The Bad will surely shoot at the Ugly if
  992. he can in round 2, since this dominates any other strategy.)  So it
  993. definitely makes a difference to the Ugly in this case to shoot at the
  994. Bad.
  995.  
  996. But all this is under the assumption that no shooter can tell what
  997. the others are about to do until after all have shot.  This isn't
  998. entirely unreasonable--we can certainly set up a game that plays
  999. this way--but suppose we assume instead:
  1000.  
  1001.   All three start out with guns initially holstered.
  1002.   Each one is a blindingly fast shot: he can grab his holstered gun,
  1003.     aim, and fire in 0.6 second.
  1004.   A shooter can redirect his unholstered gun at a different target and 
  1005.     fire in just 0.4 second.
  1006.   The reaction time of each shooter is just 0.2 second.  That is, any
  1007.     decision he makes to act can be based only on the actions of the 
  1008.     other two up to 0.2 second before he initiates his own action.
  1009.   The bullets travel between shooters in less than 0.1 second and
  1010.     stop any further action when they hit.
  1011.  
  1012. Then I *think* the conclusion holds for p=q=r=1: The best strategy is
  1013. to wait for someone else to grab for their gun, then shoot that
  1014. person, therefore nobody will shoot at anyone.  At least I haven't yet
  1015. thought of a case in which you improve your survival by shooting at
  1016. anyone.  Of course this is only good if you don't mind waiting around
  1017. the circle forever.
  1018.  
  1019. -- David Karr (karr@cs.cornell.edu)
  1020.  
  1021. In article <1993Apr5.210749.2657@cs.cornell.edu>,
  1022. karr@cs.cornell.edu (David Karr) writes: 
  1023. > In article <1993Apr1.123404.18039@vax5.cit.cornell.edu> mkt@vax5.cit.cornell.edu writes:
  1024. >>In article <1993Mar25.022459.10269@cs.cornell.edu>, karr@cs.cornell.edu (David Karr) writes:
  1025. > [...]
  1026. >>> 5. Are there any value of p, q, and r for which it is ever in the
  1027. >>>    interest of one of the shooters to fire into the ground?
  1028. >>> 
  1029. >>     Yes, p=1, q=1, r=1.  The only way for one to survive is to have the other
  1030. >>     two shoot at eachother.  Shooting at anyone has no effect on ones personal
  1031. >>     survival.
  1032.  
  1033. > I assume by "has no effect on" you mean "does not improve."
  1034. >>  If all follow the same logic, they will keep shooting into the
  1035. >>     ground and thus all live.
  1036. > I was very pleased by this answer but I had to think about it.  First
  1037. > of all, it assumes that continuing the fight forever has a positive
  1038. > value for each shooter.  My preferred assumption is that it doesn't.
  1039. > But even if each shooter is simply trying to maximize his probability
  1040. > of never being shot, I wonder about the "has no effect" statement.
  1041. > Suppose that in round 1 the Good fires into the ground and the Bad
  1042. > shoots at the Good.  Then the Ugly lives if he shoots the Bad and dies
  1043. > if he does anything else.  (The Bad will surely shoot at the Ugly if
  1044. > he can in round 2, since this dominates any other strategy.)  So it
  1045. > definitely makes a difference to the Ugly in this case to shoot at the
  1046. > Bad.
  1047.  
  1048. Here's where the clincher comes in!  If we "assume" the object of the game
  1049. is to survive, and that there exists _one_ unique method for survival, then
  1050. all the shooters will behave in the same fashion.  Obviously the above case
  1051. will not hold.  How do we distinguish between the good, the bad and the ugly?
  1052. If the command is "Shoot" then all will shoot and somebody is going to wind up
  1053. lucky (Prob that it is you is 1/3).  If the command is "No Shoot", then all
  1054. will fire into the ground (or just give up and go home--no sense waitin' around
  1055. wastin' time, ya know)...
  1056.  
  1057. But wait, you cry!  What if there exists _more than one_ solution for optimal
  1058. survival.  Then what?  Will the Good the Bad and the Ugly each randomly decide
  1059. between "Shoot" and "No Shoot" with .5 probability?  If this is true, then is
  1060. it in your best interest to shoot someone?  If it is, then we arrive back at
  1061. square one:  since we assume all shooters are geniouses, then all will shoot--
  1062. arriving at an optimal solution of "Shooting".  If the answer is "No Shooting",
  1063. we arrive at an optimal solution of "No Shooting".  If there is no effect on
  1064. your personal survival, then do we analyze this with another .5 probability
  1065. between the chances of soemone shooting or not shooting?  If the answer to this
  1066. is "Shoot" then we arrive at square one:  all will Shoot; if no, then all will 
  1067. withold.  If there is no effect, then we arrive at another .5 probability...
  1068. Obviously you can see the recursion of this process.
  1069.  
  1070. Perhaps this would be easier to discuss if we let p=1, q=1, r=0.  Obviously, in
  1071. terms of survival, shooting at the ugly would be wasting a shot.  Thus we have 
  1072. made a complex problem more simple but retaining the essence of the paradox:
  1073.  
  1074.  If there are two gunmen who shoot and think with perfect logic and are kept
  1075.  inside a room and are allowed to shoot at discrete time intervals without
  1076.  being able to "see" what your opponent will do, what will happen?
  1077.  
  1078. Let's say that you are one of the gunmen (the Good).  You reason "My probability
  1079. to survive the next round is independent on whether or not I fire at him."  So
  1080. you say to yourself, "Fire at the opponent!  I'll get to stop playing this
  1081. blasted game." But then you realize that your opponent will also think the same
  1082. way...so you might think that you might as well not shoot.  But if your
  1083. opponent thinks that way, then you know that:  1.  You can survive the next
  1084. round.  2.  You can shoot him if you wish on this round (if you like).  So you
  1085. say to yourself, "Fire at the opponent!".  But you know the opponent thinks the
  1086. same way so... you're dead.  But really, you say.  Is there a way of "knowing"
  1087. what the opponent thinks?  Of course not.  You reason that you can know your
  1088. probability of shooting your opponent (either 1 or 0).  You reason that your
  1089. opponent has a variable probability of shooting you.  Thus from your
  1090. perspective, p=1 and r<1.  We already discussed this case and said "Shoot!". 
  1091. But wait you cry!  What if the opponent figures this out too:  p<1, r=1?  Sorry,
  1092. you're both dead.  'nuff said!  This applies to the p=r=q=1 case as well.
  1093.  
  1094. > But all this is under the assumption that no shooter can tell what
  1095. > the others are about to do until after all have shot.  
  1096.  
  1097. Ay, there's the rub!
  1098.  
  1099. >This isn't entirely unreasonable--we can certainly set up a game that plays
  1100. > this way--but suppose we assume instead:
  1101. >   All three start out with guns initially holstered.
  1102. >   Each one is a blindingly fast shot: he can grab his holstered gun,
  1103. >     aim, and fire in 0.6 second.
  1104. >   A shooter can redirect his unholstered gun at a different target and 
  1105. >     fire in just 0.4 second.
  1106. >   The reaction time of each shooter is just 0.2 second.  That is, any
  1107. >     decision he makes to act can be based only on the actions of the 
  1108. >     other two up to 0.2 second before he initiates his own action.
  1109. >   The bullets travel between shooters in less than 0.1 second and
  1110. >     stop any further action when they hit.
  1111. > Then I *think* the conclusion holds for p=q=r=1: The best strategy is
  1112. > to wait for someone else to grab for their gun, then shoot that
  1113. > person, therefore nobody will shoot at anyone.  At least I haven't yet
  1114. > thought of a case in which you improve your survival by shooting at
  1115. > anyone.  Of course this is only good if you don't mind waiting around
  1116. > the circle forever.
  1117.  
  1118. Hmmn...alternate ploy:
  1119.  
  1120. 0.0  You begin to unholster your gun
  1121. 0.2  Opponents begin unholstering guns.  You aim into the ground for .2 sec.
  1122. 0.4  Opponents are unholstered you are unholstered.  They note you aren't
  1123.      aiming at them.  They haven't aimed at anyone yet.
  1124.  
  1125. What happens now?  I'll have to think about it, but I haven't seen anything
  1126. fundamentally different between this and the above case yet.
  1127.  
  1128. More ideas to consider:
  1129.  
  1130. You begin unholstering your gun but only for .1  sec (you place it by .2 )
  1131. You begin unholstering your gun but only for .09 sec (you place it by .19)
  1132.  
  1133. You start to aim for .1  sec and then stop aiming.
  1134. You start to aim for .1  sec and then turn and aim at another.
  1135. You start to aim for .09 sec and then stop aiming (or aim at another)
  1136.  
  1137. -Greg
  1138.  
  1139. Looking at the answer for decision/truel, I came across the following:
  1140.  
  1141. >Each player can obtain a positive expected payoff by shooting at one of the
  1142. >other players, and it is known that an infinite sequence of misses will
  1143. >result in a zero payoff for all players. So it is known that some player's
  1144. >strategy must involve shooting at another player rather than deliberately
  1145. >missing.
  1146.  
  1147. This may be true but it's not obvious to me.  For example, suppose A, B,
  1148. and C are passengers in a lifeboat in a storm.  If they all stay aboard,
  1149. the lifeboat is certain to sink eventually, taking all three to the 
  1150. bottom with it.  If anyone jumps overboard, the two remaining in the
  1151. boat are guaranteed to survive, while the person who jumped has a 1%
  1152. chance of survival.
  1153.  
  1154. It seems to me the lifeboat satisfies the quoted conditions, in the
  1155. sense that if nobody jumps then the payoff for all is zero, and the
  1156. payoff for jumping is 0.01 which is positive.  But it is not clear to
  1157. me that the three shouldn't just all sit still until someone goes nuts
  1158. and jumps overboard despite everything, for this strategy gives a 67%
  1159. chance of survival (assuming everyone is equally likely to "crack"
  1160. first) vs. only 1% for jumping by choice.  Even if there is a wave
  1161. about to swamp the boat, I'd wonder if the situation wouldn't just
  1162. reduce to a game of "chicken," with each person waiting until the last
  1163. minute and jumping only if it seems the other two have decided to sink
  1164. with the boat if you don't jump.
  1165.  
  1166. On the other hand, this situation is set up so it is always worse to
  1167. be the first person to jump.  In the truel I don't think this is true,
  1168. but only because of the asymmetry of the odds, and to determine
  1169. whether *anyone* shoots, it is easiest to proceed directly to
  1170. considering B's point of view.
  1171.  
  1172. Whenever it is B's turn to shoot, B can divide the possible courses of
  1173. action into four possibilities (actually there are seven, but three
  1174. are ruled out a priori by obvious optimizations of each individual's
  1175. strategy):
  1176.  
  1177. Nobody ever shoots (expected value 0)
  1178. A shoots first (at B, expected value <= .7)
  1179. C shoots first (at B, expected value <= .5)
  1180. B shoots first (at C, expected value .7)
  1181.  
  1182. In fact the value of "A shoots first" is strictly less than .7 because
  1183. in case A misses, the same four possibilities recur, and all have
  1184. expected payoff < 1.
  1185.  
  1186. So the value of "B shoots first" uniquely maximizes B's value function,
  1187. ergo B will always shoot as soon as possible.
  1188.  
  1189. The rest of the analysis then follows as in the archive.
  1190.  
  1191. -- David Karr (karr@cs.cornell.edu)
  1192.  
  1193. > Looking at the answer for decision/truel, I came across the following:
  1194. >
  1195. > >Each player can obtain a positive expected payoff by shooting at one of the
  1196. > >other players, and it is known that an infinite sequence of misses will
  1197. > >result in a zero payoff for all players. So it is known that some player's
  1198. > >strategy must involve shooting at another player rather than deliberately
  1199. > >missing.
  1200. >
  1201. > This may be true but it's not obvious to me.  For example, suppose A, B,
  1202. > and C are passengers in a lifeboat in a storm.  If they all stay aboard,
  1203. > the lifeboat is certain to sink eventually, taking all three to the 
  1204. > bottom with it.  If anyone jumps overboard, the two remaining in the
  1205. > boat are guaranteed to survive, while the person who jumped has a 1%
  1206. > chance of survival.
  1207. >
  1208. > It seems to me the lifeboat satisfies the quoted conditions, in the
  1209. > sense that if nobody jumps then the payoff for all is zero, and the
  1210. > payoff for jumping is 0.01 which is positive.  But it is not clear to
  1211. > me that the three shouldn't just all sit still until someone goes nuts
  1212. > and jumps overboard despite everything, for this strategy gives a 67%
  1213. > chance of survival (assuming everyone is equally likely to "crack"
  1214. > first) vs. only 1% for jumping by choice.  ...
  1215.  
  1216. Yes and no. Yes in the sense that if you treat the game as a psychological
  1217. one, the best strategy is as you say. But treating it as a mathematical
  1218. game, you've got to adhere to your strategy and you've got to assume that
  1219. others will adhere to theirs.
  1220.  
  1221. I.e. as a mathematical game, "Don't jump at all" and "Don't jump unless I
  1222. crack" are different strategies, and the first one is often (not always)
  1223. superior - e.g. if I take "Don't jump at all" and the others take "Don't
  1224. jump unless I crack", I'm certain to survive and the others each have a
  1225. 50.5% chance, which is better from my point of view than a 67% chance of
  1226. survival for all of us. As a psychological game, some of the mathematical
  1227. strategies may simply not be available - i.e. you cannot control what you
  1228. will do if you crack, and so we commonly use "Don't jump" to mean "Don't
  1229. jump unless I crack", since "Don't jump at all" is not an available strategy
  1230. for most real humans. But for mathematical analysis, the problem has to tell
  1231. you what strategies you are not allowed to take.
  1232.  
  1233. What the argument above shows is that "Don't jump at all" is not a stable
  1234. strategy, in the sense that if everyone takes it, it is in everyone's
  1235. interest to change strategy. I.e. it shows that someone will jump
  1236. eventually, even if it's only the result of someone actually having taken
  1237. "Don't jump unless I crack".
  1238.  
  1239. Applied to the truel, the argument above *does* show that someone's strategy
  1240. will involve shooting at another player: the strategy "Don't shoot at all"
  1241. is unstable in exactly the same way as "Don't jump at all" was. But I agree
  1242. it allows for a lot of leeway about how and when the deadlock gets broken,
  1243. and your argument showing that it is always in B's interest to shoot is more
  1244. satisfactory.
  1245.  
  1246. David Seal
  1247.  
  1248.